Determinismus-Schlupfloch?

Ich habe über die Frage nachgedacht, die ich gestern gepostet habe, und mir ist ein besserer Weg eingefallen, sie zu stellen.

Ich versuche herauszufinden, warum QM "reine Zufälligkeit" erfordert. Angenommen, Sie haben ein Photon mit einer verborgenen Variablen. Diese verborgene Variable ist ein Pseudozufallszahlengenerator F ( T ) R so dass 0 F ( T ) 1 . Wenn F ( T ) > 0,5 , geht das Photon durch den Polarisator, und wenn F ( T ) 0,5 , Es tut nicht. Wenn der Experimentator herausfinden könnte, was dieses PRNG ist, könnte er das Ergebnis jeder Messung vorhersagen, was mehr ist, als QM vorhersagen kann.

Mit anderen Worten, das Photon hat eine lokale verborgene Variable, die, wenn sie bekannt wäre, die Möglichkeit einer "echten" Zufälligkeit beseitigen würde, während sie immer noch die von QM vorhergesagte Wahrscheinlichkeitsverteilung reproduziert.

Der Satz von Bell schließt diese Möglichkeit jedoch aus. Damit habe ich kein Problem – auf Lokalität zu verzichten ist für mich in Ordnung. Also bedenken Sie Folgendes:

Der PRNG ist nicht länger eine verborgene Variable jedes Photons, sondern eine verborgene Variable eines verschränkten Zwei-Photonen-Systems. Ich bin sicher, dass dies mit einem PRNG durchgeführt werden kann, aber der Einfachheit halber nehmen wir an, dass dem gesamten System zwei einzelne PRNGs zugeordnet sind: G 1 ( T ) Und G 2 ( T ) .

Die Photonen werden verschränkt und getrennt. Photon 1 bewegt sich mit Winkel auf Polarisator 1 zu θ 1 und Photon 2 geht mit Winkel auf Polarisator 2 zu θ 2 . Es ist bekannt, dass die Wahrscheinlichkeit, dass jedes Photon die gleiche Messung liefert, gegeben ist durch:

P ( θ 1 , θ 2 ) = cos 2 ( θ 1 θ 2 )

und dies wurde experimentell verifiziert. Da die Winkel jedes Polarisators geändert werden können, während jedes Photon noch im Flug ist, ist klar zu sehen, dass es einen sofortigen Zusammenhang zwischen den Messergebnissen geben muss.

Für mich bedeutet dies jedoch immer noch keine echte Zufälligkeit.

Angenommen, Photon 1 erreicht seinen Polarisator zeitlich zuerst T 1 . Ob es durch den Polarisator geht, wird einfach durch den booleschen Wert angegeben X 1 = G 1 ( T 1 ) > 0,5 . Definieren Sie nun einen weiteren booleschen Wert

Y = G 2 ( T 2 ) < cos 2 ( θ 1 θ 2 )

, Wo T 2 ist die Zeit, zu der Photon 2 an seinem Polarisator ankommt. Ob Photon 2 den Polarisator passiert, ist dann gegeben durch:

X ¯ 1 Y ¯ + X 1 Y

Soweit ich das beurteilen kann, verletzt dies keines der Postulate von QM oder irgendeine Art von No-Go-Theorem, und es ist deterministisch. Was habe ich falsch gemacht?

Das Problem hier ist, dass Sie ausdrücklich angegeben haben, dass Photon 2 den Polarisator nicht passieren wird, wenn Photon 1 dies tut und der Winkel zwischen ihnen ist π / 2 . Das ist nicht wahr; Photon 2 kann Polarisator 2 noch passieren, aber die Messungen sind nicht mehr korreliert. Der Punkt zu sagen, dass es jetzt zufällig ist, ist, dass es keine Möglichkeit gibt, zu bestimmen, ob es den Polarisator passieren wird, ohne mit dem Photon zu interagieren. Im Wesentlichen ist es unmöglich, den PRNG vor der Messung herauszufinden

Antworten (2)

Es wurden deterministische Alternativen zur indeterministischen Kopenhagener Interpretation der Quantenmechanik veröffentlicht.

Die De Broglie-Bohm-Theorie ist die bekannteste.

David Bohm schrieb ein Buch The Undivided Universe: An Ontological Interpretation of Quantum Theory, kurz bevor er 1992 zusammen mit Basil Hiley starb. Das Buch erklärt Bohms deterministische Interpretation und vergleicht sie mit indeterministischen Interpretationen wie der Kopenhagener Interpretation und der Viele-Welten-Interpretation. Link zur Buchbesprechung . Zitat aus der Rezension: „So entwickelt sich in der Bohmschen Mechanik die Konfiguration eines Teilchensystems über eine deterministische Bewegung, die durch die Wellenfunktion choreografiert wird. Insbesondere wenn ein Teilchen in einen Doppelspaltapparat geschickt wird, der Schlitz, durch den es geht durchläuft und wo es auf der fotografischen Platte ankommt, werden vollständig durch seine Anfangsposition und Wellenfunktion bestimmt.

Die deterministische Interpretation wurde nicht widerlegt.

In der Stanford University Encyclopedia of Philosophy gibt es einen ausführlichen Artikel über Bohmsche Mechanik .

Wenn sie Vorhersagen treffen können, liefern sie beide die gleichen experimentellen Ergebnisse. Der Vorteil der Kopenhagener Sichtweise ist, dass sie auf natürliche Weise der QFT dient. Den von mir befragten Experten war keine bohmsche Interpretation der QFT bekannt.
@ Davidmh Das ist wirklich überraschend, dass die von Ihnen befragten Experten die Interpretation nicht kannten. Vielleicht kennen sie es als "Pilotwellen"-Modell oder einen anderen Begriff. Auf die Bohmsche Interpretation wird sicherlich in Peer-Review-Literatur Bezug genommen, beginnend mit Bohm, D., 1952, „A Suggested Interpretation of the Quantum Theory in Terms of ‚Hidden' Variables, I and II“, Physical Review, 85: 166–193. Eine Liste der Artikel ist hier verfügbar: bohmian-mechanics.net/research_papers.html . Auf diesem Gebiet sind Prof. Detlef Dürr von der LMU (München) und Prof. Sheldon Goldstein von Rutgers aktiv.
Es gibt ungefähr 6 Fragen zur Bohmschen Mechanik bei Physics Stack Exchange, darunter: „Warum sprechen die Leute immer noch über die Bohmsche Mechanik?“ physical.stackexchange.com/questions/7112/…
@Davidmh Es gibt "Bohmsche Mechanik und Quantenfeldtheorie" Phys.Rev.Lett. 93 (2004) 090402 arxiv.org/abs/quant-ph/0303156

Wenn Sie darauf bestehen wollen, dass eine klassische Theorie der verborgenen Variablen die Vorhersagen der QM reproduzieren kann, kann niemand sagen, dass Sie falsch liegen (nur stur). Aber wenn Sie noch etwas zugeben, dass Physik lokal sein sollte, dann haben wir Sie!

Ohne über Ihr Modell im Detail nachzudenken, kann ich also sagen, dass ein solches Modell die Vorhersagen von QM reproduzieren könnte, aber es kann nicht lokal sein.

Sie haben keinen Widerspruch gefunden, also müssen Sie nicht schlussfolgern, dass Ihre Arbeitsweise oder Annahmen falsch sind.

Ich versuche nicht, auf einer klassischen Theorie zu bestehen – ich lerne am besten, indem ich versuche, ein Gegenbeispiel zu finden und dann herauszufinden, was schief gelaufen ist (ich nehme an, man könnte es Lernen durch Widerspruch nennen?)
@Nick Sobald Sie sich von Bells Theorem überzeugt haben, sollte es keine Zweideutigkeiten darüber geben, was hier getan werden kann und was nicht. Ich würde sagen, das ist der richtige Weg.
Gut dann. Genau das mache ich hier.